HELP!!!

Yesterday I encountered this problem while I was solving a problem.But don't worry the problem has been solved.Could somebody give a solution to this: 22015a(mod20).2^{2015}\equiv a(mod{20}).You have to find 'a'.

#NumberTheory #ModularArithmetic

Note by Adarsh Kumar
6 years, 8 months ago

No vote yet
1 vote

  Easy Math Editor

This discussion board is a place to discuss our Daily Challenges and the math and science related to those challenges. Explanations are more than just a solution — they should explain the steps and thinking strategies that you used to obtain the solution. Comments should further the discussion of math and science.

When posting on Brilliant:

  • Use the emojis to react to an explanation, whether you're congratulating a job well done , or just really confused .
  • Ask specific questions about the challenge or the steps in somebody's explanation. Well-posed questions can add a lot to the discussion, but posting "I don't understand!" doesn't help anyone.
  • Try to contribute something new to the discussion, whether it is an extension, generalization or other idea related to the challenge.
  • Stay on topic — we're all here to learn more about math and science, not to hear about your favorite get-rich-quick scheme or current world events.

MarkdownAppears as
*italics* or _italics_ italics
**bold** or __bold__ bold

- bulleted
- list

  • bulleted
  • list

1. numbered
2. list

  1. numbered
  2. list
Note: you must add a full line of space before and after lists for them to show up correctly
paragraph 1

paragraph 2

paragraph 1

paragraph 2

[example link](https://brilliant.org)example link
> This is a quote
This is a quote
    # I indented these lines
    # 4 spaces, and now they show
    # up as a code block.

    print "hello world"
# I indented these lines
# 4 spaces, and now they show
# up as a code block.

print "hello world"
MathAppears as
Remember to wrap math in \( ... \) or \[ ... \] to ensure proper formatting.
2 \times 3 2×3 2 \times 3
2^{34} 234 2^{34}
a_{i-1} ai1 a_{i-1}
\frac{2}{3} 23 \frac{2}{3}
\sqrt{2} 2 \sqrt{2}
\sum_{i=1}^3 i=13 \sum_{i=1}^3
\sin \theta sinθ \sin \theta
\boxed{123} 123 \boxed{123}

Comments

@Satvik Golechha @Krishna Ar ?

Adarsh Kumar - 6 years, 8 months ago

Log in to reply

Answer's 88

Krishna Ar - 6 years, 8 months ago

Log in to reply

Yes,I actually know the answer but could you please provide a step-by-step solution?

Adarsh Kumar - 6 years, 8 months ago

Log in to reply

@Adarsh Kumar Are you joking? (Really, coz u've solved my problem...)..ANyway...ϕ(20)=8. \phi(20)=8. thus, we have that 22015(mod20)=27(mod20)2^{2015} \pmod {20} = 2^7 \pmod {20}. Which is nothing but 8!!

Krishna Ar - 6 years, 8 months ago

Log in to reply

@Krishna Ar I think you made a slight mistake as you have applied Euler's Totient theorem which can only be applied when the GCD(a,n)=1.

Adarsh Kumar - 6 years, 8 months ago

Log in to reply

@Adarsh Kumar Yes, but it still provides the answer! ^_^. I can't think of any other method.

Krishna Ar - 6 years, 8 months ago

Log in to reply

@Krishna Ar well,thanx for your help but if you can think of any other method please post!!

Adarsh Kumar - 6 years, 8 months ago

Log in to reply

@Adarsh Kumar Sure

Krishna Ar - 6 years, 8 months ago

Log in to reply

@Krishna Ar and I think that results are important but the method is more important.

Adarsh Kumar - 6 years, 8 months ago

@Krishna Ar You should always have CHINESE REMAINDER THEOREM in mind while solving this type of problems

A Former Brilliant Member - 6 years, 8 months ago

And it was this problem right?

Krishna Ar - 6 years, 8 months ago

Log in to reply

Yes,it was this problem but I figured out that the answer would not be 10 so I solved that problem but, I do not know how to find the remainder.

Adarsh Kumar - 6 years, 8 months ago

@Calvin Lin ?

Adarsh Kumar - 6 years, 8 months ago

2^2015 is a multiple of 4 And 2^2015 leaves a remainder of 3 modulo 5 ( Fermat's little theorem) so, on dividing by 20, I.e.4*5, remainder is 8 as krishna said

A Former Brilliant Member - 6 years, 8 months ago

Log in to reply

Good. Nice solution using CRT. Adarsh Kumar

Krishna Ar - 6 years, 8 months ago

yes awesome!!

Adarsh Kumar - 6 years, 8 months ago
×

Problem Loading...

Note Loading...

Set Loading...